SOLUTION: Give a polynomial g(x) so that f(x) + g(x) has a horizontal asymptote of y = 0 as x approaches positive infinity, where f(x) = \frac{2x^4 - 3x^3 - 8x^2 + 4x - 4}{x^2 + x}.

Algebra.Com
Question 1209878: Give a polynomial g(x) so that f(x) + g(x) has a horizontal asymptote of y = 0 as x approaches positive infinity, where
f(x) = \frac{2x^4 - 3x^3 - 8x^2 + 4x - 4}{x^2 + x}.

Answer by CPhill(1959)   (Show Source): You can put this solution on YOUR website!
Let $f(x) = \frac{2x^4 - 3x^3 - 8x^2 + 4x - 4}{x^2 + x}$. We want to find a polynomial $g(x)$ such that $f(x) + g(x)$ has a horizontal asymptote of $y = 0$ as $x \to \infty$.
First, we perform polynomial long division on $f(x)$:
```
2x^2 - 5x - 3
____________________
x^2+x | 2x^4 - 3x^3 - 8x^2 + 4x - 4
2x^4 + 2x^3
--------------------
-5x^3 - 8x^2
-5x^3 - 5x^2
--------------------
-3x^2 + 4x
-3x^2 - 3x
--------------------
7x - 4
```
So, $f(x) = 2x^2 - 5x - 3 + \frac{7x - 4}{x^2 + x}$.
As $x \to \infty$, the term $\frac{7x - 4}{x^2 + x}$ approaches 0.
Thus, as $x \to \infty$, $f(x)$ behaves like $2x^2 - 5x - 3$.
We want $f(x) + g(x)$ to have a horizontal asymptote of $y = 0$. This means that as $x \to \infty$, $f(x) + g(x)$ should approach 0.
So, we need $g(x)$ to cancel out the polynomial part of $f(x)$, which is $2x^2 - 5x - 3$.
Therefore, we can choose $g(x) = -(2x^2 - 5x - 3) = -2x^2 + 5x + 3$.
Then, $f(x) + g(x) = 2x^2 - 5x - 3 + \frac{7x - 4}{x^2 + x} - 2x^2 + 5x + 3 = \frac{7x - 4}{x^2 + x}$.
As $x \to \infty$, $\frac{7x - 4}{x^2 + x} \to 0$.
Thus, $f(x) + g(x)$ has a horizontal asymptote of $y = 0$ as $x \to \infty$.
Final Answer: The final answer is $\boxed{-2x^2 + 5x + 3}$

RELATED QUESTIONS

Provide an example of functions f(x) and g(x) with the limit as x approaches infinity of... (answered by richard1234)
If y=7 is a horizonal asymptote of a radical function f, then which of the following must (answered by user_dude2008)
given f(x)=4x^2-1 /x^2-16 a. Find the x-intercept(s). b. Find the y-intercept(s).... (answered by stanbon)
show by means of example that limit as x approaches a [f(x)+g(x)] may exist even though... (answered by ikleyn)
If as x approaches positive and negative infinity, the output approaches 5, we say that... (answered by MathLover1)
g(x)=f(x^3-3x^2+1) so how many extreme points y = f (x). X=0 2 3 G’(x)= -0+0-0+... (answered by ikleyn)
This is not a text book question Given F(x) = 4x^2-1/x^2 -16 a) find x intercepts b) (answered by solver91311)
given: F(x) = x/x^2-9 a. Find the x-intercept(s). b. Find the y-intercept(s). c. Find (answered by stanbon)
consider the function f(x)=7^x, g(x)=4^x, and h(x)=4^-x. the graphs of wchi functions... (answered by math_tutor2020)